Quantcast
  • Register
PhysicsOverflow is a next-generation academic platform for physicists and astronomers, including a community peer review system and a postgraduate-level discussion forum analogous to MathOverflow.

Welcome to PhysicsOverflow! PhysicsOverflow is an open platform for community peer review and graduate-level Physics discussion.

Please help promote PhysicsOverflow ads elsewhere if you like it.

News

PO is now at the Physics Department of Bielefeld University!

New printer friendly PO pages!

Migration to Bielefeld University was successful!

Please vote for this year's PhysicsOverflow ads!

Please do help out in categorising submissions. Submit a paper to PhysicsOverflow!

... see more

Tools for paper authors

Submit paper
Claim Paper Authorship

Tools for SE users

Search User
Reclaim SE Account
Request Account Merger
Nativise imported posts
Claim post (deleted users)
Import SE post

Users whose questions have been imported from Physics Stack Exchange, Theoretical Physics Stack Exchange, or any other Stack Exchange site are kindly requested to reclaim their account and not to register as a new user.

Public \(\beta\) tools

Report a bug with a feature
Request a new functionality
404 page design
Send feedback

Attributions

(propose a free ad)

Site Statistics

205 submissions , 163 unreviewed
5,047 questions , 2,200 unanswered
5,345 answers , 22,709 comments
1,470 users with positive rep
816 active unimported users
More ...

  Does Feynman path integral include discontinuous trajectories?

+ 5 like - 0 dislike
965 views

While reading this derivation of relation of Schrödinger equation to Feynman path integral, I noticed that $q_i$ can differ form $q_{i+1}$ very much, and when the limit of $N\to\infty$ is taken, there remain lots of paths, which are discontinuous (almost) everywhere — i.e. paths consisting of disconnected points.

Do I understand this wrongly? How do such discontinuous paths disappear on taking the limit? Or maybe they have zero contribution to integral?

This post imported from StackExchange Physics at 2014-05-04 11:13 (UCT), posted by SE-user Ruslan
asked May 3, 2014 in Theoretical Physics by Ruslan (85 points) [ no revision ]
Related: Paths in the path integral where Slereah notes "In phase space, things get a bit more complicated, and only discontinuous paths in phase space contribute ('Feynman Path Integrals in a Phase Space' by Berezin)."

This post imported from StackExchange Physics at 2014-05-04 11:13 (UCT), posted by SE-user Alex Nelson

1 Answer

+ 0 like - 0 dislike

The discontinuous paths do 'disappear' when you take the continuum limit. They contribute nothing to the integral in the end. In the Euclidean picture, they are suppressed by the kinetic term in $e^{-S(\phi)}$, which looks like $\sum_t \frac{(\phi(t+a) - \phi(t))^2}{a}$.

The measure you define by taking this limit is known as Wiener measure. If you're being particularly fussy, Wiener measure is defined on distributions, but has support only on the subset of distributions which are represented by continuous functions.

One of the little details that makes QFT more difficult than QM is that the fluctuations of fields are not generally suppressed in the continuum limit. In scalar field theory in 4d, the kinetic terms looks like $\sum_x \sum_\mu a^2 (\phi(x + a e^\mu) - \phi(x))^2$, so you can get exponent 2 power law singularities in the correlation function of two field values.

This post imported from StackExchange Physics at 2014-05-04 11:13 (UCT), posted by SE-user user1504
answered May 3, 2014 by user1504 (1,110 points) [ no revision ]

Your answer

Please use answers only to (at least partly) answer questions. To comment, discuss, or ask for clarification, leave a comment instead.
To mask links under text, please type your text, highlight it, and click the "link" button. You can then enter your link URL.
Please consult the FAQ for as to how to format your post.
This is the answer box; if you want to write a comment instead, please use the 'add comment' button.
Live preview (may slow down editor)   Preview
Your name to display (optional):
Privacy: Your email address will only be used for sending these notifications.
Anti-spam verification:
If you are a human please identify the position of the character covered by the symbol $\varnothing$ in the following word:
p$\hbar\varnothing$sicsOverflow
Then drag the red bullet below over the corresponding character of our banner. When you drop it there, the bullet changes to green (on slow internet connections after a few seconds).
Please complete the anti-spam verification




user contributions licensed under cc by-sa 3.0 with attribution required

Your rights
...